PT43.S3.Q01 - photons and neutrinos

JSJS Free Trial Member
edited December 2015 in Logical Reasoning 184 karma
http://7sage.com/lsat_explanations/lsat-43-section-3-question-10/
Can anyone help me about the PT43-S3-Q1?

I understand A can fill the gap but it seems more like a sufficient assumption because of the word "only". So if I negate A and financial reward is not the only incentive, the conclusion seems to be ok. Because, you know, what if financial reward is the primary incentive?
and I chose c because when I negate c, I ask myself what if the cost is more significant than the future profit? the conclusion seems to fall apart.
Many Thanks.

Comments

  • lawschooldreamslawschooldreams Free Trial Member
    22 karma
    The "only" is justified in (A) because of the author's extreme conclusion - "Or else NO ONE will engage in original development."

    For (C), it doesn't need to be that the costs incurred are INSIGNIFICANT in comparison to the financial benefit. We only need that the costs not OUTWEIGH the benefit. This is not what (C) is saying and therefore is too extreme. Tricky question! Let me know if that helps.
Sign In or Register to comment.